You are on page 1of 11

MA2108S Tutorial 5 Solution

Prepared by: LuJingyi LuoYusheng


March 2011

Section 3.1
Question 7. Let xn := 1/ ln(n + 1) for n N.
(a). Use the difinition of limit to show that lim(xn ) = 0.

Proof. Given any > 0, since

> 0, e > 1. By the Archimedean Property, K N such

1
1

1
1
that K > e 1. i.e. ln(K + 1) > 1 . i.e. ln(K+1)
< . Hence ln(1+n)
0 = ln(1+n)

1
ln(K+1)

< n K.

Hence lim(xn ) = 0.
7(b). Find a specific value of K() as required in the definition of limit for each of (i)
= 1/2, and (ii) = 1/10.
1
(i) For = 12 , take K() = 7. Then ln(K()+1)
< . Hence by part(a)

1
1
< n K().
ln(n+1) 0 < ln(K()+1)
1
(ii) For = 10
, take K() = 22026. Then

1
< n K().
ln(n+1) 0 < ln(K()+1)

1
ln(K()+1)

< . Hence by part(a)

Question 8. Prove that lim(xn ) = 0 if and only if lim(|xn |) = 0. Give an example to


show that the convergence of (|xn |) need not imply the comvergence of (xn ).
1

Proof. We can see that lim(xn ) = 0 > 0, K N such that |xn 0| = |xn | = ||xn |
0| < n K lim(|xn |) = 0. Hence lim(xn ) = 0 if and only if lim(|xn |) = 0
The convergence of (|xn |) need not imply the convergence of (xn ). Example: xn := (1)n .
Then lim(|xn |) = 1. But (xn ) does not converge.

Question 9. Show that if xn 0 for all n N and lim(xn ) = 0, then lim( xn ) = 0.


Proof. For all > 0, since 2 > 0 and lim(xn ) = 0, K N such that |xn | < 2 n K.

Since xn > 0 for all n N, |xn | = xn < 2 n K. This implies that xn < n K.

Hence by definition, lim( xn ) = 0.


Question 10. Prove that if lim(xn ) = x and if x > 0, then there exists a natural number
M such that xn > 0 for all n M .
Proof. Since lim(xn ) = x > 0, take = x > 0, then M N such that |xn x| < x n
M . Hence x x < xn < x + x n M . Hence xn > 0 n M .
Question 11. Show that lim( n1

1
)
n+1

= 0.

0 = n(n+1) = n21+n < n1 . > 0, by the

1
Archimedean Property, K N such that K1 < . Hence n1 n+1
0 < n1 < n K.

Proof. n N, observe that n1


Hence by definition, lim( n1

1
)
n+1

1
n+1

= 0.

Question 12. Show that lim( 31n ) = 0.

Proof. Observe that for all n, 31n 0 =

1
3n

1
.
2n

1
(1+2)n

1
(by
1+2n

the Bernoullis Inequality)<

Given any > 0, since 2 > 0, by the Archimedean Property, K N such that K1 < 2,

1
1
1
which implies that 2K
< . Hence 31n 0 < 2n
2K
< n K. Hence by definition,
lim( 31n ) = 0.
2

Question 15. Show that lim( nn! ) = 0.

Proof. Observe that nn! 0 =

n2
n!

<

n2
n(n1)(n2)

n2
n3 3n2 +2n

> 0, by the Archimedean Property, K N such that


1
K

n2
n3 3n2

1
K

1
= n3
n > 3.

1
< . Then nn! 0 < n3

2n1
3n2

<

< n K + 3. Hence by definition, lim( nn! ) = 0.


n

Question 16. Show that lim( 2n! ) = 0.


n

Proof. For all n 3, observe that 2n! 0 =

2n
n(n1)321

=2

2 n2
3

1
where h > 0. Hence by the Bernoullis inequality, 2
Since 32 < 1, let 23 = 1+h

1 n2
1
2
2 1+(n2)h
2 1+h
< (n2)h
if n 3.

2
Kh

> 0, by the Archimedean Property, K N such that K1 < h


,
2
2n

n
2
2
< . Hence n K + 2, n! 0 < (n2)h
kh
< . Hence lim( 2n! ) =

Given any > 0, since


which means

2 n2

h
2

0.
Question 17. If lim(xn ) = x > 0, show that there exists a natural number K such that
if n K, then 12 x < xn < 2x.
Proof. Since lim(xn ) = x > 0, take =
|xn x| < =

x
2

n K. Hence

x
2

x
2

> 0. Then by definition, K N such that

< xn < 32 x < 2x

if n K.

Section 3.2
Question 1(b). Establish either the convergence or the divergence of the sequence
X = (xn ). xn :=

(1)n n
.
n+1

Answer. X is divergent.
Proof. Suppose xn is convergent and lim(xn ) = l. Take = 12 . By definition, K N such

(1)n n

that n+1 l < 21 n K.

2n 2n

(1)2n+1 (2n+1)

1
<

l
and

l
In particular, for all n K, (1)

(2n+1)+1
< 12 . These imply
2n+1
2
2n

2n+1

that 2n+1
l < 12 and 2n+2
+ l < 12 . Hence by the triangle inequality, 12 + 21 = 1 >
2n
2n+1

+
2n + 2n+1 > 2n + 2 = 1 if n K. This implies that

l
+
l
2n+1
2n+2
2n+1
2n+2
2n+2
2n+2
1 > 1, which is a contradiction.
Hence the sequence is divergent.
3

Question 2. Give an example of two divergent sequences X and Y such that:


(a). their sum X + Y converges.
Answer. Let X := ((1)n ) and Y := ((1)n+1 ). Then X and Y diverge but X + Y = 0
converges.
(b). their product XY converges.
Answer. Let X := (0, 1, 0, 1 ) and Y := (1, 0, 1, 0 ). Then X and Y diverge but
XY = 0 converges.

Question 3. Show that if X and Y are sequences such that X and X + Y are
convergent, then Y is convergent.
Proof. Since X and X + Y are convergent sequences. By limit theorem, Y = (X + Y ) X
also converges.
Question 4. Show that if X and Y are sequences such that X converges to x 6= 0 and
XY converges, then Y converges.
Proof. Claim: there exists a K such that for all n K, xn 6= 0.
Proof of Claim:
Case 1: x > 0. Then take = x > 0. Since lim(X) = x, by definition, K N such that
|xn x| < x n K. Hence xn > x x = 0 n K.
Case 2: x < 0. Then take = x > 0. Since lim(X) = x, by definition, K N such that
|xn x| < x n K. Hence xn < x x = 0 n K.
Hence K N such that xn 6= 0 n K. Since XY converges, let lim(XY ) = z.
Consider the K-tail of X, Y, XY , the K-tail of X converges to x and the K-tail of XY
converges to z. Since Y =
z
.
x

XY
X

if X 6= 0, by limit theorem, the K-tail of Y converges to

Hence Y converges.

Question 5. Show that the following sequences are not convergent.


4

(a). (2n )
Proof. According to Ex1.13, 2n > n n N.
Suppose to the contrary, lim(2n ) = l exists. Then take = 1, by definition,

K N

such that |2n l| < 1 n K. Hence n < 2n < l + 1 n K.


Since l + 1 R, by the Archimedean Property, H N such that H > l + 1. Hence n
max{H, K}, 2n > n > l + 1. This is a contradiction. Hence (2n ) is not convergent.
(b). ((1)n n2 )
Proof. Suppose to the countrary, lim((1)n n2 ) = l exists. Take = 12 , by definition,
K N such that |(1)n n2 l| <
In particular, |(1)2n 4n2 l| <
that |4n2 l| <

1
2

1
2

1
2

n K.

and |(1)2n+1 (2n + 1)2 l| <

and |4k 2 + 4k + 1 + l| <

1
2

1
2

n K. These imply

n K. Hence by the triangle inequality,

1 > |4n2 l| + |4k 2 + 4k + 1 + l| |8k 2 + 4k + 1| > 1 n K. This implies that 1 > 1,


which is a contradiction.
Hence ((1)n n2 ) is divergent.
Question 6. Find the limits of the following sequences.

(b). lim

(1)n
n+2

1
1
Also, lim n+2
= lim n+2
= 0.

n
Hence by the Squeeze Theorem, lim (1)
= 0.
n+2

Answer. Since n+2

(d). lim

n+1

n n

1
.
n+2

Answer. lim

(1)n
n+2

n+1

n n

= lim

n
1

+ n
n n
n

= lim

1
n

+ lim

1
n

lim

1
n

= 0.

Question 7. If (bn ) is a bounded sequence and lim(an ) = 0, show that lim(an bn ) = 0.


Explain why Theorem 3.2.3 cannot be used.

Proof. Since (bn ) is bounded, there exists M > 0 such that |bn | M
> 0, since lim(an ) = 0, and

n N. Given any

> 0, by definition, K N such that |an | <

n K.

Hence |an bn | < |an |M < n K.


Hence lim(an bn ) = 0. Alternatively, it also follows from the squeeze theorem since M |an |
|bn | M |an | for all n.
Theorem 3.2.3 cannot be used because (bn ) may not be convergent.

Question 9. Let yn :=

n+1

n for n N. Show that (yn ) and ( nyn ) converge.

Find their limits.

1
1
Answer. Observe that yn = n + 1 n = n+1+
. Hence 0 n+1+
1n .
n
n

Since lim(0) = lim 1n = 0, by the Squeeze Theorem, lim(yn ) = 0.
p

1
Observe that nyn =
= n(n+1)
n(n + 1) n = n
= 1 1 . Hence
n(n+1)+n
1+ n +1
+1
n

1
1
q
lim( nyn ) =
= 2.
1
1+lim( n
)+1
Question 10. Determine the following limits.

(a). lim((3 n)1/2n ).


Answer. Note that by Example 3.1.11(c), if c > 0, then lim(c1/n ) = 1. By Example
3.1.11(d), lim(n1/n ) = 1.
1/4n

lim((4n)1/4n ) = 1
Hence, lim((3 n)1/2n ) = lim(31/2n ) lim(n1/4n ) = lim(31/2n ) lim 14
1 1 = 1.
(b). lim((n + 1)1/ ln(n+1) ).
Answer. Let (n + 1)1/ ln(n+1) = a, then n + 1 = aln(n+1) . Hence a = e.
Hence lim((n + 1)1/ ln(n+1) ) = lim(e) = e.
Question 12 If a > 0, b > 0, show that lim (

Proof.
6

(n + a)(n + b) n) =

a+b
.
2

lim (

(n+a)(n+b)n
(n + a)(n + b) n) = lim (
) = lim ( (a+b)n+ab

= lim (

a+b+ ab
n
a
(1+ n
)(1+ nb )+1

)=

(n+a)(n+b)+n
lim (a+b)+lim ab
n

= a+b
.
2
lim (1+ a )lim (1+ b )+1
n

(n+a)(n+b)+n

Question 13 Use the Squeeze Theorem 3.2.7 to determine the limits of the following.
1

(a) (n n2 )

(b) ((n!) n2 )

Proof.
(a) Since n nn for all n,
1

then 1 (n) n2 (nn ) n2 = n n .


1

But lim 1 = lim (n n ) = 1,


1

hence by the Squeeze Theorem, we have lim ((n) n2 )) = 1.


(b) Since n! nn for all n,
1

then 1 (n!) n2 (nn ) n2 = n n .


1

But lim 1 = lim (n n ) = 1,


1

hence by the Squeeze Theorem, we have lim ((n!) n2 )) = 1.

Question 14 Show that if zn := (an + bn ) n where 0 < a < b, then lim (zn ) = b.
Proof.
Since 0 < a < b,
then bn an + bn 2 bn ,
1

then (bn ) n (an + bn ) n (2 bn ) n .


1

But lim (bn ) n = b = lim 2 n lim (bn ) n = lim (2 bn ) n ,


1

hence by the Squeeze Theorem, we have lim (an + bn ) n = b,


i.e. lim (zn ) = b.

Question 15 Apply theorem 3.2.11 to the following sequences, where a, b satisfy


0 < a < 1, b > 1.
n

(a) (an )

(b) ( 2bn )

(c) ( bnn )

(d) ( 232n )

3n

Proof.
n+1

(a) Since an > 0 n, and lim aan = a < 1,


hence by Theorem 3.2.11, we have lim (an ) = 0.
(b) Since
but

b
2

bn
2n

bn+1
2n+1
bn
2n

> 0 n, and lim

= 2b ,

can be either greater or smaller than 1,

hence Theorem 3.2.11 does not apply here.


(c) Since

n
bn

> 0 n, and lim

n+1
bn+1
n
bn

1
b

< 1,

hence by Theorem 3.2.11, we have lim ( bnn ) = 0.


(a) Since

23n
32n

> 0 n, and lim

23(n+1)
32(n+1)
23n
32n

8
9

< 1,
3n

hence by Theorem 3.2.11, we have lim ( 232n ) = 0.

Question 16
(a) Give an example of a convergent sequence (xn ) of positive numbers with
= 1.
lim xxn+1
n
(b) Give an example of a divergent sequence with this property.
Answer.
(a) Consider the convergent sequence (xn ) = ( n1 ).
n
) = lim n+1
= 1.
We have lim (xn ) = 0 and lim ( xxn+1
n

(b) Consider the divergent sequence (xn ) = (n).


We have (xn ) is not bounded and hence diverge, and
lim ( xxn+1
) = lim n+1
= 1.
n
n
Question 17 Let X = (xn ) be a sequence of positive real numbers such that
lim ( xxn+1
) = L > 1. Show that X is not a bounded sequence and hence is not
n
convergent.

P roof.
8

Since lim ( xxn+1


) > 1,
n
then > 1 and N1 N, such that

xn+1
xn

> n N1 .

Hence xN1 +k k xN1 , k N.


Let be any given positive real number.
Since (k ) is unbounded,
hence N2 N, such that k >

xN1

k N2 .

Then n N1 + N2 , xn nN1 xN1 >

xN1

xN1 = .

Therefore, X is not a bounded sequence and hence not convergent.

Question 18 Discuss the convergence of the following sequences, where a, b satisfy


0 < a < 1, b > 1.
n

(b) ( nb 2 )

(a) (n2 an )
n

(c) ( bn! )

(d) ( nn!n )

P roof.
2 n+1

a
(a) Since n2 an > 0 n, and lim (n+1)
n2 an

= (lim (1 + n1 ))2 a = a < 1,

hence by Theorem 3.2.11, we have lim (an ) = 0.


(b) Since

bn
n2

> 0 n, and lim

bn+1
(n+1)2
bn
n2

= (lim (1

1
))2
n+1

b = b > 1,

hence by Question 17, we have ( nb 2 ) diverges.


(c) Since

bn
n!

> 0 n, and lim

bn+1
(n+1)!
bn
n!

b
= 0 < 1,
= lim n+1
n

hence by Theorem 3.2.11, we have lim ( bn! ) = 0.


(d) Since

n!
nn

> 0 n, and lim

(n+1)!
(n+1)n+1
n!
nn

n n
= lim ( n+1
) =

1
1 n
lim (1+ n
)

= e1 < 1,

hence by Theorem 3.2.11, we have lim ( nn!n ) = 0.

Question 19 Let (xn ) be a sequence of positive real numbers such that


1

lim (xn n ) = L < 1. Show that there exists a number r with 0 < r < 1
such that 0 < xn < rn for all sufficiently large n N. Use this to show that
lim (xn ) = 0.
9

P roof.
1

Since lim (xn n ) = L < 1,


1

then r < 1 and N N, such that xn n < r n N.


Hence 0 xn < rn n N,
but lim 0 = lim (rn : n N ) = 0.
Hence, by squeeze theorem, we have lim (xn : n N ) = 0.
Hence lim (xn ) = lim (xn : n N ) = 0.

Question 20
(a) Give an example of a convergent sequence (xn ) of positive numbers with
1

lim xn n = 1.

(b) Give an example of a divergent sequence with this property.


Answer.
(a) Consider the convergent sequence (xn ) = ( n1 ).
1

We have lim (xn ) = 0 and lim xn n = lim ( n1 ) n = 1.


(b) Consider the divergent sequence (xn ) = (n).
We have (xn ) is not bounded and hence diverge, and
1

lim xn n = lim n n = 1.

Question 21 Suppose that (xn ) is a convergent sequence and (yn ) is such that
for any > 0 there exists M such that |xn yn | < for all n M.
Does it follow that (yn ) is convergent?
P roof.
The answer is yes.
Let x := lim (xn ), > 0,
then N1 N, such that |xn x| < 2 , n N1 .
By the assumption, N2 N, such that |yn xn | < 2 , n N2 .
10

Hence, |yn x| |yn xn | + |xn x| <

= , n max{N1 , N2 },

i.e. (yn ) converges (to x).

Question 22 Show that if (xn ) and (yn ) are convergent sequences, then the sequences
(un ) and (vn ) defined by un := max{xn , yn } and vn := min{xn , yn } are also convergent.
P roof.
Notice thatun = 21 (xn + yn + |xn yn |) and yn = 21 (xn + yn |xn yn |),
n N by Exercise 2.2.16.
Since (xn ) and (yn ) are convergent sequences,
lim (un ) = lim ( 12 (xn + yn + |xn yn |)) = 12 (lim (xn ) + lim (yn ) + | lim (xn ) lim (yn )|),
and lim(vn ) = lim( 12 (xn + yn |xn yn |)) = 12 (lim(xn ) + lim(yn ) | lim(xn ) lim(yn )|).
Hence, (un ) and (vn ) are also convergent.

11

You might also like